LSAT and Law School Admissions Forum

Get expert LSAT preparation and law school admissions advice from PowerScore Test Preparation.

 Administrator
PowerScore Staff
  • PowerScore Staff
  • Posts: 8919
  • Joined: Feb 02, 2011
|
#22768
Complete Question Explanation

Must Be True-SN. The correct answer choice is (B)

This stimulus contains several statements that invite inferences. In the first sentence, we see that public criticism of a socially necessary service leads to government regulation. The third sentence just applies this to child care services: because child care services are a socially necessary service and are publicly criticized, they will be regulated.

Answer Choices (A) is wrong because nothing in the stimulus addresses improving the quality of services. (A) assume that government intervention improves the quality. This may or may not be true.

Answer Choice (B): This is the correct answer choice. The second sentence in the stimulus contains a broad statement: when an activity is regulated by the government, it becomes more expensive. We can apply this principle to what we learned from the third statement and determine that childcare services will become more expensive.

Answer choice (C) is wrong because nothing in the stimulus addresses improving the quality of services. This answer choice assumes that government intervention improves the quality. This may or may not be true.

Answer Choice (D), at first glance, seems like a good answer choice because it is consistent with the first sentence. However, the first sentence only talks about socially necessary services, not policy in general. (D), then, goes too far.

Answer Choice (E) makes a logical error, a mistaken negation: we know that if child-care services are regulated, the cost will increase. But, we do not know anything if they are not regulated. Child care costs might increase anyway for some reason completely unrelated to regulation.
 ellenb
  • Posts: 260
  • Joined: Oct 22, 2012
|
#11908
Dear Powerscore,

I got this question right, I just want to make sure that I understood it correctly.

So, can we make conditional statements out of the statements given for this question:

We have:

Public criticism
and ----> Government Regulation---> Expensive
Socially Necessary Service

Ex: Childcare

Childcare Public criticised
and ----> Childcare Government Regulation---> Childcare Expensive
Childcare Socially Necessary Service

Thus, we can conclude that Childcare will get more expense.


Thus, when it said in the explanations "we can make some inferences from the statements" did they mean that we can put them in the conditional reasoning statements just like I did above?

Thanks in advance!

Ellen
User avatar
 Dave Killoran
PowerScore Staff
  • PowerScore Staff
  • Posts: 5850
  • Joined: Mar 25, 2011
|
#11910
Hi Ellen,

Yes, the phrase "we can make some inferences from the statements" is simply there to indicate that there are some inferences created by the statements above. They don't always have to be conditional in nature, but in this case they are since this stimulus contains conditional reasoning.

Thanks!
 ellenb
  • Posts: 260
  • Joined: Oct 22, 2012
|
#11917
Thanks Dave, is my diagram correct based on the statements in the stimulus?

Regards,

Ellen
 Steve Stein
PowerScore Staff
  • PowerScore Staff
  • Posts: 1153
  • Joined: Apr 11, 2011
|
#11928
Hi Ellen,

Yes, your diagram is correct based on the statements in the stimulus.

Hope that's helpful!

~Steve
 gat2114
  • Posts: 1
  • Joined: Jul 03, 2018
|
#47385
Hi powerscore,

I would like to ask why is the sufficient condition "criticise" and the necessary condition "regulate"?
Personally, I thought that it should be the other way round since if something has been regulated, it would have been criticised. No?

Thank you,
G
 Adam Tyson
PowerScore Staff
  • PowerScore Staff
  • Posts: 5153
  • Joined: Apr 14, 2011
|
#47437
You've got that backwards, gat2114! In the stimulus we see the necessary conditioner indicator "only" ("Governments have only one response"). That "only" is referring to the response, which is regulation, and not to the criticism. In other words, criticism is not the only thing; regulation is the only thing. That's one way that we can determine that criticism is sufficient and regulation is necessary, rather than the other way around.

Another approach you could take is to paraphrase the argument. Is the author saying if the government gets criticized, then they will regulate? Or is it saying if the government regulates something, then there will be criticism? I think you'll find it looks more more like the former than the latter. And while chronology is not implied by a conditional relationship, it certainly makes more sense here for the regulation to be the necessary condition since it is a response to the criticism, rather than preceding it.

Through the use of indicator words, paraphrasing, and a dash of some common sense in holistically analyzing the stimulus, we can often make sense of conditional relationships that at first seem confusing. Use all the tools in your toolbox!
 Juanq42
  • Posts: 29
  • Joined: Jul 21, 2019
|
#67328
I successfully narrowed my contenders to B and E, but ultimately chose E (and not thoroughly reading it).

In eliminating B, I highlighted the level of certainty as too strong... but after reviewing the stimulus, I tried to find the phrase/words that support the absolute answer. Does the final phrase, "the government is certain to respond" support the degree of certainty? Are there other phrases in the stimulus that indicate - without a doubt - that the cost of providing childcare will indeed increase?

Thank you!
 James Finch
PowerScore Staff
  • PowerScore Staff
  • Posts: 943
  • Joined: Sep 06, 2017
|
#67343
Hi Juan,

This is a Must Be True question, meaning we're looking for a statement we know to be true 100%. The easiest way to get to that 100% certainty is with conditional reasoning, so that's what we need to look out for in a question like this. Here we're given that whenever a government responds to public criticism of a socially necessary activity, it regulates that activity, which then leads to that activity becoming more expensive, or:

GRPCSNA :arrow: RSNA :arrow: SNA More Expensive

Then we're told that childcare is a socially necessary activity that is being criticized, which means that the government will respond to that criticism. This gives us our sufficient condition in the A :arrow: C chain that we've already made, allowing us to infer that first, the government will regulate childcare, and second, that childcare will become more expensive as a result of this regulation. So we should look for either of these inferences as a Prephrase our answer choice.

(B) fits the bill, giving us the A :arrow: C inference, while (E) gives us a Mistaken Negation of the second conditional statement given.

Hope this clears things up!
User avatar
 Hosseingold29
  • Posts: 5
  • Joined: Oct 29, 2022
|
#98293
hey i think u have made a mistake . u have supposed that , regulating = responding

however who has said this is right ?!!!
I think this q isnt true basicly :hmm:

Get the most out of your LSAT Prep Plus subscription.

Analyze and track your performance with our Testing and Analytics Package.